• Shuffle
    Toggle On
    Toggle Off
  • Alphabetize
    Toggle On
    Toggle Off
  • Front First
    Toggle On
    Toggle Off
  • Both Sides
    Toggle On
    Toggle Off
  • Read
    Toggle On
    Toggle Off
Reading...
Front

Card Range To Study

through

image

Play button

image

Play button

image

Progress

1/112

Click to flip

Use LEFT and RIGHT arrow keys to navigate between flashcards;

Use UP and DOWN arrow keys to flip the card;

H to show hint;

A reads text to speech;

112 Cards in this Set

  • Front
  • Back

Fact pattern: development with a provision restricting something (size or something) - deeds are all recorded, but later, part of the development is sold off without the provision about the restriction mentioned in the new deed and someone tries to do something (make smaller lots, or paint a house a different color or something). This is an implied reciprocal servitude - even if the new buyer doesn't have actual notice, he has constructive notice due to the advertising of the common scheme and the provision that is in all the other deeds in the development. He also has inquiry notice due to the common characteristics of all the other lots or houses. Note that an injunction is an ok remedy!



A developer purchased a 60-acre parcel of wooded land and divided the parcel into 20 three-acre lots. The developer advertised the rustic character of the lots and the intent to sell the lots for development as single-family residences. This was in conformity with the zoning restrictions on the land, which required that the land be used for residential purposes and that the size of each lot not be less than two acres. Over a period of several years, the developer sold 15 of the lots. The deed for each of these lots contained the following provision: This deed is subject to the condition that the property may only be used for residential purposes and may not be subdivided but must be sold in its entirety. This condition shall be a covenant running with the land and shall be binding on all owners, their heirs, devisees, successors, and assignees. The deed for each lot was promptly and properly recorded. The developer, facing financial difficulty, sold the remaining five lots to a land speculator. The deeds to these lots did not contain the character and size provision that the developer had inserted into the other deeds, nor did the speculator have actual knowledge of the developer’s advertising related to the character and size of the lots. The land speculator, acting in response to a zoning change that reduced the minimum permissible size of a lot to only one acre, has obtained governmental approval to divide each of the five remaining lots in thirds and is now offering the 15 lots for sale. An owner of one of the three-acre lots has brought suit against the speculator seeking an injunction to prevent him from selling the lots in less than three-acre parcels. Can the speculator successfully defend against this lawsuit?

answer c (or 3) is correct: what to know -


Answer choice C is correct. Each of the speculator’s lots is subject to an implied reciprocal servitude. Although normally an equitable servitude must be in writing, the existence of a common scheme is evidenced by the developer’s advertizing and the insertion of the character and size provision into the deeds for the other 75% of lots. In addition, although the speculator had neither actual nor record notice of the size restrictions, the speculator had inquiry notice based on the uniformity in size of each of the other lots that had been sold. Answer choice A is incorrect because, as the name suggests, an implied reciprocal servitude does not require record notice, but is implied from the existence of a common scheme. Answer choice B is incorrect because obtaining governmental approval by complying with the applicable zoning ordinance does not protect an owner from also having to adhere to a private land use restriction. Answer choice D is incorrect because, although the speculator did not purchase the lots for the purpose of constructing a residence on each lot, the speculator did not use the property for nonresidential purposes.



does an option to purchase land (with consideration) have to be in writing?

yes, one of the real estate type contracts that is subject to the SOF

can a mortgagee compell the owner of a warehouse to fix a roof over an unused part of the warehouse if the owner is not in default?

yes - owner's waste impairs mortgagee's security interest

A man whose terminally ill aunt had promised to devise an undeveloped parcel of land to him sold the parcel to a friend. The friend purchased the property based on the nephew’s false assertion that he owned the parcel; the friend was unaware of the aunt’s ownership of the parcel. The friend did not perform a title search and did not record the deed, which was a general warranty deed. Subsequently, the aunt died. As promised, she devised the parcel to her nephew. The personal representative of the estate executed and recorded a deed transferring title to the parcel to the nephew. After the aunt’s death, no one paid the property taxes on the parcel. Eventually, the state seized the parcel and sold it through a tax sale. Before expiration of the redemption period that is statutorily permitted to the owner of the real property, the friend learned of the sale of the parcel for delinquent taxes. Claiming ownership of the parcel, the friend sought to pay the delinquent taxes and other costs and fees associated with the sale and thereby redeem the parcel. The buyer of the parcel at the tax sale, who had no prior knowledge of the friend’s claim with respect to the parcel, objected. In an action to determine ownership of the parcel, if the court finds for the friend, what is the likely reason?

Answer choice D is correct. Because the friends rights vest when the property is transferred to the nephew. When a person who attempts to transfer real property that he does not own subsequently becomes owner of that property, the after-acquired title doctrine provides that title to the property automatically vests in the transferee. Answer choice A is incorrect because, while the nephew did record his deed, the friend did not. Consequently, the nephew, not the friend, was owner of record of the parcel. Answer choice B is incorrect because although the friend presumably did not have actual knowledge that the aunt owned the parcel, the friend was on constructive notice that the nephew did not own the property. Moreover, even assuming the friend’s status was that of a good faith purchaser, this status is irrelevant in ascertaining the friend’s rights in relationship to the subsequent buyer. Answer choice C is incorrect because the doctrine of equitable conversion, which treats a buyer as the equitable owner of property during the period between execution of the contract and delivery of the deed, is irrelevant.

42 - tricky: when a buyer buys a new house and fails to get an inspectiona and doesn't get an express warranty and the house gets cracks in the foundation, who wins?


1. The builder, because the builder did not give the buyer a written warranty as to the condition of the house.
2. The builder, because the buyer failed to conduct a home inspection.
3. The buyer, because the builder of a residence is liable for any material defects in the house.
4. The buyer, because the builder breached the warranty of fitness or suitability.

4. is the answer


Answer choice 3. is incorrect because a builder is not responsible for any material defect in the house, but only those that could not have been uncovered by the buyer during a reasonable inspection. (so, even thought he buyer didn't inspect, this was the type of defect that would not have been uncovered at an inspection - if it has bee the type that an inspection could have uncovered, the buyer is SOL)

* An individual acquired a newly constructed house with a purchase money mortgage. Although the deed was recorded, through an oversight by the mortgagee, the mortgage was not. Several years later, the individual sold the house at its fair market value to a couple who obtained a purchase money mortgage through another mortgagee. Both the deed and the mortgage were recorded. Neither the couple nor the second mortgagee was aware of the prior mortgage. Shortly thereafter, the couple was killed in an accident, survived by their two young children. The couple did not leave a will. Under the law of intestate succession, the young children are the rightful heirs of their parents. The children’s financial guardian, having been contacted by both mortgagees, has filed an appropriate action to determine ownership of the house. The jurisdiction is a lien state with regard to mortgages. In addition, the applicable recording act reads, “No conveyance or mortgage of real property shall be good against subsequent purchasers for value and without notice unless the same be recorded according to law.” Who is entitled to priority with respect to the house?

Answer choice C is correct. The recording statute is a notice statute. Since the second mortgage was given by the mortgagee without notice as to the first mortgage, it has priority over the first mortgage. A purchase money mortgagee is treated as having paid value for purposes of the recording act. In addition, since the second mortgage was given by the couple before their deaths, the children’s claims to the property as heirs of their parents are junior to the second mortgage. The recording act would not be helpful to the children’s claim because the second mortgage was recorded before their interests arose and because as heirs they are not purchasers for value protected by the act. With regard to the first mortgage, while the children as heirs are not protected by the act, they are protected under the shelter rule because their parents were entitled to priority over the first mortgage by virtue of acquiring the property without notice of that mortgage. For all these reasons, answer choices A, B, and D are incorrect.

* A widower owned a residence in fee simple absolute. He contracted to sell it to a couple. The couple did not record the contract. The contract did not require either party to acquire or maintain casualty insurance on the premises, and neither party did so. After the parties entered into the contract, the widower continued to occupy the residence. A week before closing, the residence was completely destroyed by a fire caused by a lightning strike. On whom does the risk of loss fall?

Answer choice B is correct. Under the doctrine of equitable conversion, the couple became equitable owners of the property upon the execution of the contract of sale. As such, they bear the risk of loss. Answer choice A is incorrect because the determination of the party who bears the risk of loss does not turn on whether the contract is recorded. Answer choice C is incorrect because, unless the contract specifies otherwise, neither party has a duty to insure the property. Answer choice D is incorrect because, under the majority rule, the risk of loss is not determined by possession of the property.



NOTE - not in NY!!! D is correct in NY.

* In order to finance the purchase of a property, the buyer received a loan and in return gave the lender a promissory note secured by a mortgage on the property. Subsequently, the buyer divided the property into two parcels, retaining one of the parcels and selling the other to a friend. The friend took the parcel subject to the mortgage. The buyer and the friend agreed that each would be liable for one-half of the outstanding mortgage. One year later the buyer disappeared. Since the buyer was no longer paying one-half of the mortgage obligation, the lender threatened to foreclose on the property. The friend paid off the outstanding balance of the loan. The applicable jurisdiction recognizes the lien theory of mortgages. Can the friend bring a foreclosure action against the buyer’s parcel?

Answer choice A is correct. Since the friend paid the buyer’s loan obligation in full in order to protect his own interest, the friend became subrogated to the lender’s rights based not only on the personal obligation of the buyer, but also on the mortgage on the land itself. Answer choice B is incorrect because the way in which the friend acquired his interest in his own parcel is irrelevant to the issue of subrogation. Answer choice C is incorrect because, while the friend does not have an ownership interest in the buyer’s parcel, the friend may nevertheless seek to foreclose on the mortgage on that parcel, which it holds through its right of subrogation. Answer choice D is incorrect because, although payment of another person’s obligation that is secured by a mortgage, by a person who is under a legal duty to do so, can give rise to the right of subrogation, payment of such an obligation by a person in order to protect his property interest is also entitled to subrogation.



Look up subrogation!

taking subject to the mortgage!!! I get it now! If I buy a 500k property and the mortgage is 300k and I buy it subject to the mortgage, I pay the 500k and the bank gets its 300k and the seller gets the rest!!! I assume no mortgage. If the property is upside down (I pay 500k and the mortgage is 600k), then the title is encumbered and we got problems.

-

* A mother made a gift of unimproved real property to her son. The son promptly and properly recorded the deed, but did not inspect the property nor otherwise make use of it by building structures or making other improvements. The son, however, did pay the real estate taxes imposed on the property. Subsequently, the mother, forgetting about her conveyance of the property, sold it at its fair market value. The buyer promptly and properly recorded the deed. The buyer, who was not aware of the son’s ownership of the property, began to construct a house on the property. Upon learning about the buyer’s construction activities, the son, unaware of his mother’s transaction with the buyer, brought an appropriate legal action to halt the buyer’s activities and declare title to the property. Will the buyer be successful in defending against the son’s lawsuit?

Answer choice C is correct. Regardless of the type of recording act that governs, the grantee of real property is protected from a subsequent purchaser’s claims of ownership to the property by recording his deed prior to the subsequent conveyance. Answer choice A is incorrect because, although the recording act does not protect a subsequent grantee who is a donee from an unrecorded deed, it does protect any grantee, including a donee, who records his deed prior to the subsequent conveyance. Answer choice B is incorrect because there is no requirement that the owner of real property make productive use of the property. Answer choice D is incorrect because the son’s payment of the real estate taxes on the property is irrelevant to a priority dispute between the son and a subsequent purchaser.



I GOT THIS WRONG, but it's a an easy answer: hard and fast rule - if you record the deed, subsequent buyers have constructive notice and the first buyer who recorded the deed wins!!!!

* The fee simple owner of land devised it to a private educational institution “for so long as the land herein conveyed is used for educational purposes; if the land is not so used, then to my daughter and her heirs.” At the time of the owner’s death, the owner’s spouse was deceased and the owner’s only two children, a son and a daughter, were alive. The owner devised all of his other real property interests to his son. The daughter died shortly after her father, devising her real property’s interests to her only child, who was alive at the time of her death. Immediately after the owner’s death, the institution constructed a classroom building on the land and has held classes in the building each year thereafter. Thirty years after the owner’s death, the educational institution seeks to sell the land to a developer who intends to construct single-family homes on the land. Both the son and daughter’s child, who are the owner’s only living heirs, are alive. The applicable jurisdiction has adopted the following statute: “A nonvested property interest is invalid unless when the interest is created, it is certain to vest or terminate no later than 21 years after the death of an individual then alive, or the interest either vests or terminates within 90 years after its creation.” The applicable jurisdiction does not impose time limitations on the exercise of interests that follow a defeasible fee property interest. In order to convey marketable title to the developer, whom must the institution convince to agree to the transfer?

Answer choice B is correct. Upon the owner’s death, the institution had a fee simple subject to an executory interest in the land, and the daughter had an executory interest. Under the common law Rule Against Perpetuities the daughter’s interest could, and in fact would, vest more than 21 years after her death. In such case, her interest would be stricken, and the son, having been devised the owner’s other real property interests, would hold a possibility of reverter in the land. However, the jurisdiction has adopted the “wait and see” approach to the Rule. Since the daughter’s interest did vest within 90 years of its creation, it does not violate the jurisdiction’s Rule. An executory interest may be devised, and the daughter devised her executory interest to her child. Since the condition imposed on the institution (i.e., use of the property for educational purpose) would be violated by the developer, the property would then automatically pass to the daughter’s child. Consequently, the institution must secure her approval for the transfer in order to convey marketable title. For these reasons, answer choices A, C, and D are incorrect.

* In order to purchase undeveloped land, the buyer sought a 10-year loan from a third-party lender. The buyer executed a promissory note and mortgage on the property. The lender promptly and properly recorded the mortgage. As part of the transaction, the lender also required the buyer to execute a quitclaim deed to the property to the lender, which the buyer was to give to an independent escrow agent. Under the terms of the escrow arrangement, the agent was to record the quitclaim deed to the lender upon notification that the buyer had defaulted on the loan. The escrow agreement also provided that, upon recording, the buyer’s rights in the property would cease. (this type of agreement is void as to public policy - Valerie's note) The buyer made installment payments on the loan, as required by its terms, for two years, but subsequently was unable to make the required loan payments. The lender notified the escrow agent of the buyer’s default and the escrow agent recorded the quitclaim deed. The lender, choosing not to foreclose on the mortgage, has advertised the property for sale at an amount significantly higher than the outstanding balance in the mortgage. Shortly thereafter, the buyer, receiving a sizeable inheritance, offered to pay the lender the full amount of the outstanding mortgage debt, which was more than 85% of the original mortgage loan. The lender refused to accept the buyer’s payment. The buyer has filed an action to compel the lender to accept the payment, release the mortgage, and to void the quitclaim deed. The applicable jurisdiction has the following statute: “No conveyance or mortgage of real property shall be good against subsequent purchasers for value and without notice unless the same be recorded according to law.” Who should prevail?

Answer choice B is correct. The escrow arrangement seeks to circumvent the buyer-mortgagor’s equitable right to redeem the mortgaged property. The court will strike down the escrow arrangement and permit the buyer to redeem the mortgaged property. Answer choice A is incorrect because the lender’s knowledge and the applicability of the recording statute are irrelevant. Although the lender obviously had notice of the prior mortgage since the lender was the mortgagee, the lender did not challenge the priority of the mortgage, but merely refused to foreclose on the property based on the mortgage. Answer choice C is incorrect because the lender has obtained title to the property by means of an arrangement that seeks to prevent the buyer-mortgagor from exercising the equitable right of redemption. As a consequence, the court will void the lender’s title. Answer choice D is incorrect because a mortgagor’s equitable right of redemption does not depend on the amount of the original mortgage loan that the mortgagor has paid.



This type of agreement if void as against public policy!!!!! know this!!!!

* The owner of an undeveloped lot agrees to sell the lot to a buyer. The written agreement identifies the parties, describes the property in sufficient detail, specifies the price to be paid, and spells out the payment terms. The agreement is signed by the owner. In accord with the agreement, the buyer pays the required down payment to the owner. Subsequently, the buyer constructs a garage on the lot as the first step towards building a three-story residence, but, due to a financial reversal, abandons his construction efforts. May the seller bring an action to compel the buyer to complete the purchase?

Answer choice C is correct. The buyer’s actions of paying a portion of the purchase price to the owner and constructing a garage on the lot (which constitutes a substantial improvement of the lot) are persuasive evidence that a contract between the parties exists. The doctrine of part performance may be asserted by either party to a land sale contract in order compel specific performance of the contract. Answer choice A is incorrect because, although the buyer did not sign the written agreement and otherwise would enjoy the protection of the Statute of Frauds, the buyer’s actions with respect to the lot are sufficient to establish the existence of the contract through part performance. Answer choice B is incorrect because, although the owner’s remedy at law (damages) is adequate, the owner is permitted to seek specific performance under the theory of mutualities of remedies. Answer choice D is incorrect because the facts do not indicate that the seller detrimentally relied on the buyer’s agreement to purchase the property in a manner that would create a hardship.

I got this wrong, but it's a simple rule:


In order to purchase her residence, a homeowner gave a lender a promissory note in exchange for a loan. The note was secured by a mortgage on the residence. Five years later, the homeowner gave a second lender a promissory note in exchange for a loan, in order to add another room to the residence. This note was also secured by a mortgage on the residence. Three years later, the homeowner gave a third lender a promissory note in exchange for a loan in order to construct a deck on the residence. This note was also secured by a mortgage on the residence. Each mortgage was properly recorded promptly after execution. Recently, the homeowner has failed to make timely payments with regard to the first mortgage. The first lender has declared the homeowner in default and, in accord with the terms of the mortgage, accelerated the obligation. The first lender forecloses on the mortgage. At the foreclosure sale, the third lender purchases the residence. To which of the following mortgages is the residence now subject?


Answer Choices: 1. Only the first mortgage.
2. Only the second mortgage.
3. Both the first and the second mortgages.
4. Neither the first nor the second mortgages.

Answer choice D is correct. The general rule is that the purchase of property at a foreclosure sale eliminates not only the mortgage being foreclosed but also all junior mortgages on the property. This rule applies even when the purchaser is one of the junior mortgagees. For this reason, answer choices A, B, and C are incorrect.

if I send an acceptance and it gets lost in the mail, then I send a rejection, what result?



what might invalidate that result?



if the rejection is sent prior to the acceptance, is a contract formed?



if the acceptance is sent first, is a contract formed even if the offeror only gets the rejection and not the acceptance? generally, yes (except when the offeror relies on the rejection - then the offeree is estopped from bringing asserting the existance of a contract)

generally, the mailbox rule says that the acceptance is valid under these circumstances



if the offeror gets the rejection and not the acceptance and then relies on the rejection in some way, this invalidates what would have been a valid contract



no.



generally, yes (except when the offeror relies on the rejection - then the offeree is estopped from bringing asserting the existance of a contract)


is the D's atty in a fraud case allowed to impeach a witness on cross by asking about a prior arrest for writing bad checks when the witness was never charged with that crime?



can a witness who is not the D be cross examined about that witness' prior bad acts?

no - not charged, so this does not show that the witness was untruthful (this isn't a prior bad act - being arrested and not even charged shows that the cops made a mistake and nothing more!)



a witness may sometimes be asked on cross-examination about a prior bad act. There are some limitations, however: the judge must determine that the prior bad act is probative of untruthfulness, and the attorney must have a good faith basis for asking about the conduct.





when a private golf course knows that balls frequently fly out of their property onto a public road or waterway, is this a public nuisance?



if a ball flies out and hits a child navigating in a sailboat on the waterway nearby, does the boy have a cause of action against the golf course for public nuisance?

yes - the nuisance is to the public



yes - the boy suffered a special injury that is different in kind to that suffered by the general public. this is a classic public nuisance.

* The president of a closely held corporation personally purchased an inn from a seller. In addition to a small down payment, the president executed a note for the remainder of the purchase price. The note was secured by a mortgage on the property. The mortgage and related deed were timely and properly recorded. The note contained a due-on-sale clause, which required the president to obtain the seller’s consent in order to transfer the inn. The president transferred the inn to her corporation without the seller’s permission or knowledge. For several years, the president continued to make timely payments on the note from her personal bank account in order to conceal the transfer from the seller, until shortly before the president filed for personal bankruptcy, at which time the seller learned of the transfer. In the bankruptcy proceeding, the note is subject to discharge unless the seller’s failure to exercise his rights under the due-on-sale clause is due to intentional misrepresentation by the president. The current value of the inn is less than the outstanding balance owed on the note. Does the president’s conduct constitute intentional misrepresentation?

Answer choice C is correct. A misrepresentation can arise from nondisclosure by a person who is under a duty to disclose. A misrepresentation can also arise from conduct and from the concealment of a fact. A person may generally rely on a misstatement unless the falsity of the misstatement is obvious. Here, the president had a duty to disclose based on the terms of the due-on-sale clause. Answer choice A is incorrect because, as noted, a misrepresentation need not be in writing. Answer choice B is incorrect because a person is not under a duty to investigate the truthfulness of a misrepresentation. Answer choice D is incorrect because intentional misrepresentation is not confined to real property transactions, but can arise in any type of transaction in which the plaintiff suffers actual economic (i.e., pecuniary) loss due to the defendant’s misrepresentation.

are leading questions allowed on direct?



when you call a witness for your side and the witness starts to behave in a hostile manner (evading the question and answering differently than you expect), can you ask a leading question?



when can you ask leading questions of your own witness?

generally, no



the federal rules allow leading questions on direct examination in certain situations when it is necessary to develop testimony, such as when the witness is a child, has difficulty communicating due to age or a physical or mental problem, is hostile, is an adverse party, or is associated with an adverse party.



a party generally may not ask leading questions of his witnesses unless the testimony is purely related to uncontested or incidental matters or when it is necessary to develop the witness’s testimony for one of the reasons mentioned above.

Under the MPC, does a conspiracy require an overt act? by whom?



MPC adopts a unilateral conspiracy theory, does this mean you can be charged with conspiracy without making an agreement to commit a crime?



under the MPC, how is the scope of conspiracy narrowing for a co-conspirator than under the majority?



is voluntary intoxication a defense to conspiracy?

yes; by either conspirator; if your co-conspirator commits one, you are deemed to have committed one



no; in order for conspiracy to exist, there has to be an agreement (a unilateral conspiracy is one in which an undercover cop pretends to conspire and you go along with it)



under the Model Penal Code, a member of a conspiracy is not liable for a crime unless he aids and abets in its commission. This differs from the majority rule, in which a conspirator can be convicted of both the offense of conspiracy and all substantive crimes committed by any other co-conspirators acting in furtherance of the conspiracy.



conspiracy is a specific intent crime for which voluntary intoxication may be a defense.



do you have a right to notice and a hearing prior to termination of your welfare benefits or will a hearing after suffice?

before is required - otherwise unconstitutional

If a retailer orders 10 widgets to be sent in 5 installments and the manufacturer ships the first installment along with an extra term (that the manufacturer retained a security interest in all items until the contract was paid in full) and the retailer pays for the first installment and then cancels the rest, can the manufacturer sue for breach? does it matter that the two sides are merchants?



does the added term materially affect the contract?



what is the "knockout rule?" (doesn't apply here, but just so I know)

yes, manufacturer can sue retailer for breach; a contract was formed - the additional term cannot be enforced, but the retailer can't get out of the contract just because of the additional term; no, it doesn't matter



yes; it materially affects the contract, but it isn't enforceable, so it doesn't void the contract



applies to conflicting terms proposed by merchant parties (battle of the forms) and "knocks them out" effectively cancelling them out and saving the other terms of the contract

buyer wants to buy a house because a famous person allegedly had owned it at one point; buyer asks seller about the history of the house and seller says, "I don't know." Buyer buys the house anyway, then finds out that the house was never owned by a famous person. What result?

unilateral mistake - no duty on the part of the seller and the buyer can't void the contract to buy the house

* A witness to an armed robbery identified a suspect in a proper police lineup that was not attended by the suspect’s attorney. Charges were brought against the suspect, but the witness, a tourist from out of the country, had returned to her home country before the trial began. At trial, the prosecutor seeks to introduce the witness’s prior statement of identification into evidence. The defendant objects to the introduction of the evidence. Should the court allow the prior statement of identification into evidence?
*

* Rationale: Answer choice D is correct. While a prior, out-of-court statement of identification is often considered non-hearsay and can be admissible as substantive evidence, the witness who made the prior statement of identification must have testified at the present trial and have been subject to cross-examination concerning the identification for it to be admissible.
* while some hearsay exceptions do depend on the unavailability of the witness, prior statements of identification are considered nonhearsay, and are not exceptions to the hearsay rule. Answer choice C is incorrect because there is no requirement that the suspect or defendant be represented by counsel at a lineup for the statement to be considered nonhearsay

does immunity granted in a criminal trial extend to a civil case?



does double jeopardy?



what is the difference between transactional and derivitive-use immunity?

no - immunity can't be granted for civil liability



no



transactional = related to the current trial; derivitive-use = in subsequent criminal trials



If I buy jerseys to take a bunch of kids to a big football game that is supposed to be held in Tampa and I have the name of the city embroidered on them (that's my contract with the seller), then the game is moved to Miami, can I get my deposit back and cancel the contract?

Yes, frustration of purpose - unexpected event destroys one party's purpose for the contract - I get my money back and I can cancel (note that the maker of the shirts didn't start making them in this case or there'd have been a reliance issue).

* On a winter day, a youth, seeking refuge from the cold, entered a small neighborhood grocery store without the knowledge of the store’s owner, who was standing at the cash register. Shortly thereafter, the only other person in the store approached the register and requested an item located on a shelf behind the owner. As the owner turned to retrieve the requested item, the individual drew a gun and commanded the owner to give him the money in the register. As the owner turned back toward the customer, the customer fired the gun at her and missed. The owner grappled with the customer and succeeded in knocking the gun out of the customer’s hand. As the customer retrieved his gun, the owner grabbed her own gun, for which she had a valid license. They fired at each other, each missing the other. Although the owner’s actions did not create an unreasonable risk of harm to the youth, the bullet from the owner’s gun nevertheless struck and killed the youth. The estate of the youth filed a wrongful death action against the store owner. Who will prevail?
* Learner Selected Answer: The store owner, because the owner acted in self-defense.

Answer choice B is correct. When a person using force in self-defense unintentionally harms an innocent third party, the person is not liable for injuries to the third party that occur while the person is acting in self-defense, so long as those injuries were accidental, rather than deliberate, and the person was not negligent with respect to the third party. Because this was the case here, answer choice B is correct, and answer choice A is incorrect.

Is passing out political leaflets in a mall speech that is protected by the Constitution?

no - this is not a public forum, even though it's open to the public

if a state statute allows the death penalty for crimes that are "heinous or brutal" will it be deemed unconstitutional?



is "premeditation" the only acceptable criterion for the death penalty?

yes, it will because it has been deemed to violate the 8th A. as "vague"



no, this is not the only acceptable statutory criterion for the death penalty

* In a criminal trial for arson, a prosecution witness testifies under oath that she saw the defendant set fire to the victim’s home. The defendant’s attorney does not cross-examine the witness, but seeks to admit testimony that the witness gave at a deposition several months before the trial. At the deposition, the witness testified under oath that she did not see the defendant set fire to the victim’s home. Should the court admit the deposition testimony?

Under Federal Rule of Evidence 801(d), a prior inconsistent statement, which otherwise would qualify as hearsay, is treated as non-hearsay. A prior inconsistent statement may be used to impeach a witness. Further, if a prior inconsistent statement made under oath at a trial, hearing, or deposition, it is admissible both to impeach the declarant’s credibility and as substantive evidence, so long as the witness testifies at the present trial or hearing and is subject to cross-examination concerning the statement. The witness need not actually be cross-examined, so long as she is subject to cross-examination at the present trial. Here, even though the defendant’s attorney did not cross-examine the witness, he had the opportunity to do so. Therefore, the testimony can be used both as substantive evidence and for impeachment purposes.

what is equitable conversion?

it's the period between the contract and the closing during which the buyer is deemed to be the owner of the property

* The owner of an electronics store brought a civil suit for the value of stolen electronics against one of his former employees, who had previously been convicted in a criminal court for the theft of the same goods. During the civil trial, the plaintiff-owner called a witness whom he hoped would testify that she saw the defendant in possession of the stolen goods the day after the electronics store was robbed. The witness, however, testified that she did not see the defendant in possession of the goods, and that she was actually out of town the day after the robbery. The plaintiff seeks to introduce the witness’s testimony in the criminal case, in which she testified that she saw the defendant in possession of the goods the next day. The defendant objects to the introduction of the statement. Should the court allow the testimony into evidence?

Answer choice C is correct. Prior inconsistent testimony is admissible as both impeachment evidence and as substantive evidence. A prior inconsistent statement made under oath at a trial, hearing, or deposition is considered nonhearsay, and is admissible to impeach the declarant’s credibility and as substantive evidence. The witness’s credibility may be called into question by showing that the witness has previously made a statement that is materially inconsistent with some part of her current testimony. Further, when that statement was made under oath, it may also be used to prove the truth of the matter asserted—in this case, that the defendant was in possession of the goods on the day after the robbery. Because answer choice C is the only choice that reflects that the testimony may be used for both purposes, answer choices A, B, and D are incorrect.

how do we describe the behavior of the tortfeasor who commits IIED?



what about assault, battery, or trespass to chattels (throwing tacks out on the race track in front of a cyclist to make them lose the race)

extreme and outrageous



willful, wanton, reckless or malicious


does this disclaimer effectively negate the implied warranty of fitness for a particular purpose: “There are no implied warranties provided with this product other than the general warranty of merchantability?" (even if the merchant's employee recommends a certain product for a particular purpose)?

yes - if in writing and displayed conspicuously on the form

A 15-year-old male was being tried in state court as an adult for murder. At voir dire, the prosecutor exercised all of his peremptory challenges to exclude persons under the age of 30 from the jury. The defendant’s attorney timely raised the issue as to whether the prosecutor had utilized his peremptory challenges in an unconstitutional manner. In response to questioning by the court, the prosecutor stated that it was his intent to exclude persons who, because of their age, would be sympathetic to the defendant. The judge found that the prosecutor’s reason was genuine and not pretextual. Should the judge sustain the defendant’s objection?


* Yes, because the prosecutor’s use of peremptory challenges violates the defendant’s Sixth Amendment right to trial by an impartial jury.
* Yes, because the defendant was a member of the affected class.
* No, because the prosecutor is permitted to exercise peremptory challenges for any rational reason.
* No, because the prosecutor’s use of peremptory challenges does not violate the Equal Protection Clause.

Rationale: Answer choice D is correct. A prosecutor is free to exercise peremptory challenges in any manner he sees fit unless such exercise violates the Equal Protection Clause. The Equal Protection Clause prevents the use of peremptory challenges for racially or gender motivated reasons. Since the prosecutor did not use peremptory challenges for either of these reasons, the judge should overrule the defendant’s objection.



NOTE that C is wrong.

* Concerned with protecting the use of federal funds from the deleterious effects of bribery, Congress enacted a statute criminalizing the acceptance of a bribe by a state or local official where the state or local government received at least $10,000 in federal funds. A county government, in exchange for its agreement to permit the housing of federal prisoners in the county’s jail, received a payment of federal funds for each prisoner. The total amount received by the county government for housing federal prisoners exceeded $100,000 annually. A federal prisoner housed in the county jail agreed to transfer title to a pickup truck to a prison guard in exchange for the guard permitting the prisoner to receive illegal conjugal visits. The prison guard was charged with violating the statute. Is the application of the statute to the guard’s taking title to the prisoner’s truck constitutional?

Answer choice C is correct. Congress may enact legislation that is necessary and proper to execute its spending power. Here the criminalization of the taking of bribes by an official of an entity that receives substantial federal funds is necessary and proper to ensure that such funds are properly spent. Such funds are provided to the state and local governments pursuant to the Spending Clause.

modified comparative negligence?

i) If the plaintiff is less at fault than the defendant, then the plaintiff’s recovery is reduced by his percentage of fault, just as in a pure comparative-fault jurisdiction;


ii) If the plaintiff is more at fault than the defendant, then the plaintiff’s recovery is barred, just as in a contributory-negligence jurisdiction;


iii) In the vast majority of modified comparative-fault jurisdictions, if the plaintiff and the defendant are found to be equally at fault, then theplaintiff recovers 50% of his total damages. In a few modified comparative-fault jurisdictions, the plaintiff recovers nothing when the jury finds that the plaintiff and the defendant are equally at fault.

what is exoneration (regarding wills)?

the concept of exoneration applies when a specific devise is subject to an encumbrance.



A specific devisee of encumbered property is not entitled to have the debt paid off by the residual estate unless the testator’s intent to do so is clear in the will. A testator can specifically require that a lien be exonerated, in which case the encumbered property will not abate to exonerate the lien unless specifically stated in the will. Absent a clause specifically exonerating the lien, the beneficiary will take subject to the lien.


Note that a general directive to pay debts is insufficient as a directive to have liens exonerated through residuary funds.

* A man decided to master the art of throwing knives. He practiced for several years, until he had the perfected his skills and was able to hit a spot no larger than a dime with confidence. After demonstrating his prowess to a friend, the man convinced the friend to stand against a wall while the man threw knives at her. The man threw three knives extremely close to the friend, but the fourth knife struck the friend, injuring her slightly. Although the friend’s injury was minor, unbeknownst to the man, she had a rare blood disorder that caused her to bleed to death. The crimes below are listed in ascending order of seriousness. What is the most serious common law crime for which the man can be convicted?

Answer choice D is correct. The man may be convicted of depraved heart murder. Depraved heart murder is a killing that results when the defendant recklessly acts with extreme indifference to human life. For this type of murder, the man need not have had the intent to cause either death or serious bodily injury. The woman’s consent to the act that led to her death is not a defense. Nor is the fact that the woman’s death would not have happened but for her rare medical condition, or the man’s lack of awareness of that condition. Because the man may be convicted of murder, the less serious crimes listed in answer choices A, B, and C are incorrect.



THINK RUSSIAN ROULETTE

what are the elements of IIED?

To sustain a claim of intentional infliction of emotional distress, a plaintiff must prove that she suffered severe emotional distress as a result of the defendant’s conduct, that the defendant intended to cause severe emotional distress, or acted with recklessness as to the risk of causing such distress, and that the defendant’s conduct was extreme or outrageous.

If someone slips in my store and gets injured and I have an employee write down a record of what happened in case the injured person sues me, is this record later admissible evidence? why not?

It's hearsay and doesn't fall under the category of any exception. Hearsay is an out-of-court statement, whether written or oral, offered to prove the truth of the matter asserted. Hearsay is not admissible unless it falls within an exception. The record in this case meets the hearsay definition and should not be admitted unless a specific exception applies. One possible applicable exception to the hearsay rule would be the business record exception, which provides that a record or other writing made in the course of regularly conducted business is admissible. However, records prepared in anticipation of litigation are not admissible as business records.



NOTE: business records that meet the definition of the exception to the hearsay rules are self-authenticating - no one needs to be available to testify about them or authenticate them

if I trip over your docile tiger, are you strictly liable for my injuries? if I freak out and fall?



is the rule different for licensees? trespassors?

no, only for the dangerous propensities of the wild animal that cause my injuries; my fearful reaction to the animal is an exception - yes you'd be strictly liable. NOTE: this question might have come out differently if the call of the question was about negligence.



the exception to strict liability for a possessor of a wild animal applies only to a trespasser who is injured by the wild animal on the possessor’s property. It does not extend to a licensee, such as a social guest, or an invitee.

what is the MPC test for a D who is not guilty by reason of mental disease or defect?

Under the Model Penal Code test, a defendant is not guilty when, at the time of the conduct, as a result of a mental disease or defect, he did not have substantial capacity to appreciate the wrongfulness of the act or to conform his conduct to the law.

what if I slip up and give my work product to the opposing counsel? did I waive the atty-client privilege? what do I have to do?

In a federal proceeding, the disclosure of a protected communication does not operate as a waiver if (i) the disclosure was inadvertent, (ii) the holder of the privilege took reasonable steps to prevent disclosure, and (iii) the holder promptly took reasonable steps to rectify the error, including contacting the party to whom the communication was disclosed and requesting that they return, sequester, or destroy the information. In this case, the facts indicate that the disclosure was inadvertent and that the attorney had acted diligently during discovery.

what is an implied-in-law contract "quasi-contract":



what is an implied-in-fact contract:

an implied-in-law contract ("quasi-contract") when one party confers a benefit on another and has a reasonable expectation of compensation. Otherwise, the benefited party would be unjustly enriched. Such situations occur when: (i) the plaintiff has conferred a "measurable benefit" on the defendant; (ii) the plaintiff acted without gratuitous intent; and (iii) it would be unfair to let the defendant retain the benefit because either the defendant had an opportunity to decline the benefit but knowingly accepted it, or the plaintiff had a reasonable excuse for not giving the defendant such opportunity, usually because of an emergency. (this is the injured horse wanders onto a neighbor's property and the neighbor cares for it story)



created when conduct indicates assent or agreement (such as when I go into Supercuts for a haircut and sit down and let them cut my hair - I've agreed to pay them, whether I indicate that I will or not)

Police officers have a reasonable suspicion, but not probable cause, that the defendant committed a robbery. The police officers, acting without a warrant, went to the defendant’s home and requested that he come to the stationhouse for fingerprinting. The defendant refused until the police officers threatened him with arrest.

the fingerprints are not admissible because they were seized in an illegal manner - can't compel him to come to the police station absent probable cause - reasonable suspicion is not enough

* A middle-aged farmer who lived by himself in a rural area had surgery to correct an orthopedic problem. Since his recovery would take about a year, he contacted a retired nurse about serving as his caretaker. While the farmer was still in the hospital, the two reached an agreement, the terms of which were specified in two letters. The letter written by the nurse identified the farmer by name and stated, “I agree to take care of your medical needs for a period of one year, starting when you leave the hospital.” The letter written by the man identified the nurse by name and stated, “I agree to pay you $10,000 per month.” Each letter was signed by its drafter. Before his discharge from the hospital, the man found out that the hospital had a less expensive program for home care, and cancelled the contract. Unable to find other employment, the nurse brought a breach of contract action against the man. Based solely on the letters, will the nurse be able to establish the existence of a contract?

Answer choice C is correct. Since the contract cannot be performed within one year from the time of the contract’s making, it is subject to the Statute of Frauds. The writings are not sufficient under the Statute of Frauds because, although together they state the essential terms of the bargain and each is signed by the promisor, neither writing references the other. In order to satisfy the Statute of Frauds, at least one of the writings must reference the other. Because the farmer’s letter does not indicate the subject matter of the contract (i.e., why the farmer is paying the nurse $10,000 a month), the nurse will be unable to enforce the agreement against the farmer. Answer choice A is incorrect because, although this agreement can be performed within one year of the beginning of performance, it is subject to the Statute of Frauds because it cannot be performed within one year of the date of its making. Answer choice B is incorrect because, although together the writings contain the essential terms of the agreement, neither writing references the other. Answer choice D is incorrect because the writings do not have to establish the existence of a valid offer and acceptance, but instead must state the essential terms of the agreement and be signed by the party against whom enforcement is sought.

* A small restaurant utilized the same beverage supplier for many years. The owner of the restaurant had developed a very strong working relationship with the supplier’s employees. The contract between the supplier and the restaurant only obligated the supplier to provide beverages, but the supplier’s employees frequently performed repairs on the restaurant’s soda dispensers, and were available to fill emergency orders even late at night and on weekends. Furthermore, the restaurant received a commission for each beverage sold at its restaurants, which the beverage supplier, unlike many other suppliers, paid in cash. The restaurant received lucrative offers to switch to other distributors, but repeatedly chose to stay with its supplier because of the personalized service. The beverage supplier decided that it would be advantageous to concentrate its business solely on larger accounts. Without obtaining approval from the restaurant, the supplier “assigned all rights and delegated all duties” arising under its contract with the restaurant to a second beverage distributor. The second distributor did not pay any consideration for this transfer. While there was no anti-assignment clause its contract with the original supplier, the restaurant was not pleased with the assignment, and refused service from the new distributor. The new distributor filed suit against the restaurant, claiming breach of contract. What is the restaurant’s strongest argument in defense?

Answer choice D is correct. A delegation of contractual duties is not permitted when the other party to the contract has a substantial interest in having the delegor perform the contract. This exception is normally invoked in contracts involving personal services. While this is not such a contract, the restaurant may argue that it repeatedly spurned more lucrative offers because it strongly preferred the service provided by the original supplier. Answer choice A is incorrect because delegation of duties is permitted in most circumstances. Answer choice B is incorrect because no consideration is necessary for an assignment or delegation to be valid. However, the presence or absence of consideration is relevant to the revocability of the assignment (an assignment supported by consideration is irrevocable, while one without consideration is not). Answer choice C is incorrect because parties to an assignment/delegation are not required to obtain the consent of a party to the original contract unless the contract specifically requires such consent.

* An employee at a toy store intervened in a dispute between two unrelated customers, a mother and a grandfather, over who was entitled to a particular hard-to-come-by doll, which was the only remaining one at the store. The employee arbitrarily determined that the mother had possession of the doll first and awarded her the right to purchase the doll. When the grandfather protested the employee’s decision, the mother threatened to inflict physical harm on the grandfather and raised her arm to strike him. Fearful that the mother would do so, the grandfather looked to the employee for help. The employee, who because of his size could easily have forestalled the mother’s attack, simply shrugged his shoulders. Before the mother made contact with the grandfather, he crumpled to floor, the victim of a stroke caused by the mother’s threat. The grandfather initiates a lawsuit against the mother, the employee, and the owner of the store on the grounds of assault for damages attributable to his stroke. The owner of the store moves to dismiss the complaint against herself for failure to state a cause of action. How should the court rule?

Answer choice C is correct. The store owner is not vicariously liable for an assault committed by one customer upon another. Any liability that the owner might have from the incident would arise from her employee’s negligence for failure to prevent the assault. Answer choice D is incorrect because, while an employer is generally not liable for an employee’s intentional torts, an employer may be liable for such a tort if the employer authorizes the employee’s use of force or force is inherent in the nature of the employment. Moreover, it was the mother, not the employee, who committed an assault against the grandfather. Answer choice A is incorrect because, while the grandfather was reasonably apprehensive of an imminent battery by the mother, neither the employee, nor vicariously the owner, committed an assault with respect to the grandfather. Answer choice B is incorrect because neither the employee nor the owner is liable for the grandfather’s injuries on the basis of assault.

If I have witness who saw someone who looks like my husband, but isn't sure, can her testimony be admitted in a civil case where I'm trying to show that my husband has been missing for 5 years and presumed dead?

All relevant evidence is admissible, unless excluded by a rule or law. Relevant evidence is that which has any tendency to make any fact of consequence more or less probable than it would be without the evidence. So, relevant evidence isn't necessarily probative, just makes increases or decreases the likelihood that a material fact is true.

what is the "bursting bubble" approach?

Under the “bursting bubble” approach followed by the federal rules and a majority of common-law jurisdictions, a presumption is not evidence in a civil case, but a preliminary assumption of fact that “bursts” after the introduction of evidence to sustain a contrary finding. Even speculative testimony may be deemed sufficient to "burst the bubble" of a presumption.

is a restaurant that serves cake made by a bakery that has slivers of glass in it strictly liable for injury?

yes - manufacturer defect and the restaurant is a commercial supplier


(not for the failure to warn, though, because this was not a foreseeable risk)

* An individual acquired a newly constructed house with a purchase money mortgage. Although the deed was recorded, through an oversight by the mortgagee, the mortgage was not. Several years later, the individual sold the house at its fair market value to a couple who obtained a purchase money mortgage through another mortgagee. Both the deed and the mortgage were recorded. Neither the couple nor the second mortgagee was aware of the prior mortgage. Shortly thereafter, the couple was killed in an accident, survived by their two young children. The couple did not leave a will. Under the law of intestate succession, the young children are the rightful heirs of their parents. The children’s financial guardian, having been contacted by both mortgagees, has filed an appropriate action to determine ownership of the house. The jurisdiction is a lien state with regard to mortgages. In addition, the applicable recording act reads, “No conveyance or mortgage of real property shall be good against subsequent purchasers for value and without notice unless the same be recorded according to law.” Who is entitled to priority with respect to the house?

Answer choice C is correct. The recording statute is a notice statute. Since the second mortgage was given by the mortgagee without notice as to the first mortgage, it has priority over the first mortgage. A purchase money mortgagee is treated as having paid value for purposes of the recording act. In addition, since the second mortgage was given by the couple before their deaths, the children’s claims to the property as heirs of their parents are junior to the second mortgage. The recording act would not be helpful to the children’s claim because the second mortgage was recorded before their interests arose and because as heirs they are not purchasers for value protected by the act. With regard to the first mortgage, while the children as heirs are not protected by the act, they are protected under the shelter rule because their parents were entitled to priority over the first mortgage by virtue of acquiring the property without notice of that mortgage. For all these reasons, answer choices A, B, and D are incorrect.

* To remedy past discrimination against women in college athletics, Congress required state colleges and universities to achieve “gender equity” in funding athletics. The percentage that each school must allocate to women’s athletics was based on a formula that took into account the average percentage of athletic department funds allocated to men's and women’s programs over a five-year period. Depending on the school’s past record, this allocation could be greater than the percentage of women enrolled in the school. A state with any schools out of compliance forfeited a percentage of its federal educational funding. A state military college first admitted women five years ago. Only 10% of its student body is currently female. To comply with the federal funding formula, the college must allocate 25% of its athletic budget to women’s sports. A male student, whose wrestling program is being discontinued because of the budget allocation, filed suit in an appropriate federal court challenging the federal law’s constitutionality. Is he likely to prevail?

Answer choice C is correct. The Fifth Amendment’s Due Process Clause includes the rights guaranteed by the Equal Protection Clause, thereby making discrimination by the federal government subject to review under the same standards as discrimination by the states. The federal statute in question discriminates based on gender, and hence “intermediate scrutiny” is the appropriate standard of review. Here, the federal government has an important interest in remedying past discrimination against women in college athletics and the law is substantially related to this interest. Both answer choices A and D are incorrect because they fail to apply the correct standard of review to the law’s gender discrimination. Answer choice B is incorrect because, even though Congress lacks the power to directly compel a state to take a specific action (i.e., allocate the budget of a state institution in a manner that favors women), Congress may use its spending power to encourage a state to take such action (here, by reducing the state institution’s federal funding if it fails to comply).

* A prosecutor called a witness to testify in a battery trial. The witness, a friend of the defendant, behaved in a hostile manner to the prosecutor, refusing to testify that the defendant was the initial aggressor in the altercation at issue, as the prosecutor believed he would. The prosecutor decided to call a second witness to minimize any damage done by the first witness’s testimony. This second witness, who was the first witness’s sister, testified that because her brother had been in a fight similar to the one in question, it was her opinion that he was not a good witness. The defendant’s attorney objects to the sister’s testimony. Should the prosecutor be allowed to ask the sister about the fight?
* No, because the altercations have no bearing on the first witness’s character for untruthfulness.
* Rationale: Answer choice D is correct. The credibility of a witness may be attacked by reputation or opinion evidence only as to the witness’s character for truthfulness or untruthfulness. Evidence that the first witness was previously in a violent altercation has no bearing on his credibility (unless he had previously denied being in such an altercation, which the facts here do not indicate). Answer choice A is incorrect because, while it contains a true statement, reputation or opinion evidence may only be used to attack a witness’s character for truthfulness. Answer choice B is incorrect because, while it is true that a witness puts his own credibility at issue by testifying, the prosecutor did not attack his credibility in the appropriate way. Answer choice C is incorrect because a party may impeach or attack the credibility of any witness, even a witness the impeaching party called.

common law burglary: what is "breaking and entering?"

opening a door is sufficient - the door doesnt need to be locked - opening a cupboard door is sufficient

Moon discounts the price of an addition for Mr. P. and begins building the addition. He expects to make $5,000 profit. When he has incurred $50,000 in expenses, Mr. P. tells him to stop working. the cost to hire another contractor to do the same work would have been $60,000. Mr. P's house is worth $25,000 more right now than it was before work began. What does Mr. P owe Moon?

normally, expectation damages would suffice (so, $50,000 plus the expected profit = $55,000). However, because Moon discounted to Mr. P, he gets restitution damages, so $60,000. The $25,000 increase in value to Mr. P's house is another measure of restitution damages, but because it is lower than the other way of calculating, we don't use that method.

can a D invoke the 5th A in a civil proceeding?



is asking to say if a handwritten note is in your writing something you can plead the 5th about?

yes, if it could be used against him in a criminal proceeding in the future



yes, if the contents of the note would incriminate you, saying the handwriting was yours is testimonial (only testimonial evidence is covered by the 5th A)

* Two defendants were on trial for conspiracy to commit robbery. The prosecution would like to introduce the testimony of a security guard at the store that was to be robbed. The security guard caught one of the defendants trying to sneak in a back entrance of the store the day before the robbery was to take place, hoping to “scope out” the store. In a panic, the defendant had said, “We haven’t even done anything wrong yet! We weren’t going to do anything until tomorrow!” The prosecutor would like to introduce this statement against both defendants as evidence of their intent to rob the store the next day. The defendants object. Against which defendant should the statement be admitted?

Answer choice A is correct. The statement is an admission by a party-opponent, which is by definition non-hearsay. Under Rule 801(d)(2)(A), a prior out-of-court statement by a party to the current litigation that is used against that party is not hearsay. As such, it is admissible against the declarant-defendant. The statement is also admissible against his co-defendant. Admissions made by one party may be imputed to another party based on certain relationships between the two, such as when the parties are co-conspirators. Although a statement by one co-party may not be imputed to another co-party solely because they are co-parties, statements made by a conspirator during and in furtherance of the conspiracy are admissible as an admission against other co-conspirators.

if I buy a stolen ring in good faith (not knowing it's stolen), do I own it (do I have title)? can the original owner get it back from me?

I own it. I'm a good faith purchaser for value. the original owner can't go after me - can go after the thief, though. (status as a merchant is irrelevant either way with this rule)

when you use pictures (for example) just to "refresh" a witness' memory, can you also admit them into evidence? can the opposing party admit them? cross-examine the witness about them?

no. yes. yes.

* A manufacturer entered into a contract with a forklift supplier to purchase 10 new forklifts for use in the manufacturer’s warehouse. The contract specified that the forklifts were to be delivered within 45 days of the execution of the contract. The day after entering into the contract, the supplier was told by a reliable source that the manufacturer was in a precarious financial position. That day the supplier, reasonably relying on the information, which was in error, sent a written notice to the manufacturer demanding assurance of the manufacturer’s ability to pay. Thirty-five days after receiving the notice, the manufacturer sent the supplier its most recent financial statements, which adequately demonstrated that the manufacturer was not in a precarious financial position and had the funds to pay for the forklifts, along with a statement of its willingness to receive the shipment of the forklifts. Immediately upon receiving the manufacturer’s correspondence the following day, the supplier called the manufacturer’s CEO and demanded his personal guarantee of payment for the forklifts before the supplier would deliver the forklifts. When the manufacturer’s CEO declined, the supplier refused to deliver the forklifts. The manufacturer then purchased forklifts from another distributor at a higher price. The manufacturer sued the forklift supplier for breach of contract. Should the manufacturer prevail?
* Learner Selected Answer: Yes, because the supplier breached the contract by failing to deliver the forklifts.

Answer choice A is correct. The supplier failed to perform its contractual obligation to deliver the forklifts to the manufacturer. Upon the supplier’s breach, the manufacturer sought cover by purchasing the forklifts from another source at a higher price. The manufacturer can recover from the supplier the difference between the cover price and the contract price. Under the UCC, either party can demand assurance of performance if there are reasonable grounds for insecurity about the other party’s ability or willingness to perform. Once such assurances are requested, performance may be suspended until they are provided. Failure to give adequate assurances within a reasonable time, not exceeding 30 days, can be treated as repudiation. Here, the manufacturer failed to provide assurances within 30 days, so the supplier was within its rights to reject the assurances and repudiate the contract. However, the supplier did not cancel the contract or take any action in reliance on the failure to timely provide assurances. Consequently, when the manufacturer provided the supplier with such assurances and retracted its implied repudiation of the contract by requesting delivery of the forklifts, the supplier was contractually obligated to the deliver the forklifts.

what is equitable conversion?

on the MBE (but not in NY), a buyer of real property is the owner as of the date of the contract, regardless of who occupies it

does fraudulent conduct negate consent? (in a rape case, for example)?

getting a girl to have sex with you by convincing her that it will cure her pelvic pain is not enought to add up to rape - she is still willing

when are punitives available for trespass to chattels?

when the conduct is wilfull or wanton (shooting at your neighbor's car cause you're pissed)

* An appropriate representative of a resident in a state mental health facility sued an official of the facility in federal court. The resident, who remained a citizen of another state, sought an injunction to compel the official to comply with state law regarding the use of the least restrictive environment approach for the care of the mentally ill. The official moved to dismiss the action as unconstitutional under the Eleventh Amendment. Should the court grant the official’s motion?

Answer choice D is correct. The Eleventh Amendment prohibits an action by a citizen of one state against another state when the basis for the action is the violation of state law. Answer choice A is incorrect because, although the plaintiff seeks an injunction rather than damages, the Eleventh Amendment prohibits the action nonetheless, as it is based on state law. Similarly, answer choice B is incorrect because, although the action is brought against the state official rather than the state, the Eleventh Amendment prohibits an action based on state law. Answer choice C is incorrect because, although a plaintiff can pursue an action despite the Eleventh Amendment in order to prevent the enforcement of an unconstitutional state statute, the plaintiff is not advancing such an argument here. Since the plaintiff is not challenging the constitutionality of the state statute under the Due Process Clause of the Fourteenth Amendment, the issue of whether the plaintiff is asserting a fundamental right is irrelevant.

when is extrinsic evidence to a contract not admissible under the Parole Evidence Rule regardless of whether the contract is integrated?

when the evidence directly contradicts what's in the contract

* In a civil trial regarding a store owner’s negligence in clearing ice from his front walkway, the defendant store owner testified on direct examination that he had cleared the ice from his front walkway on the morning of the plaintiff’s injury. On cross-examination, the plaintiff’s attorney asked the defendant if he was sure that he cleared all of the ice off the walkway, without missing any, and the defendant replied that he had. The plaintiff’s attorney then sought permission to ask the defendant about two prior incidents in the last three winters in which the defendant’s customers claimed to have fallen on patches of ice that the defendant failed to clear. The defendant’s attorney objected to the introduction of this evidence. Should the court allow the plaintiff’s attorney to question the defendant about the prior incidents?

A witness can be impeached with prior bad acts that bear upon truthfulness; failing to clear ice from a walkway has no bearing on truthfulness. Generally, a witness also can be impeached with evidence that contradicts a material part of his testimony, but nothing about the plaintiff’s questions would contradict the defendant’s testimony. Evidence of prior bad acts generally cannot be used as evidence that a witness/defendant acted in a particular way on the occasion in question. Answer choice A is incorrect because, while evidence of a person’s habit is admissible to prove conduct in conformity with the habit on a particular occasion, there is no evidence here that the defendant has a habit of not clearing ice from his walkway. The fact that there have been t

when is evidence of juvenile convictions not admissible?



is evidence of a criminal conviction the same as evidence of a "prior bad act?"



can a witness be impeached (for truthfulness) by evidence of a juvenile conviction for writing a bad check?

in a civil case


The court may occasionally permit evidence of a juvenile adjudication of a witness other than the accused under certain circumstances, but only in a criminal trial.



apparently not! Subject to certain rules, a witness may always be impeached with evidence that he has been convicted of a crime - not so with other prior bad acts.



no. if the conviction were of an adult, yes. a witness may be impeached with evidence that she has been convicted of any crime involving dishonesty or false statements within 10 years of the conviction.

does an oral promise to a charity to make a donation constitute a contract when the charity does not rely on the promise?

no - it has to be in writing for the exception for charities to take effect - other promises to make a gift can only be enforced if they are in writing and the promisee relies on the promise to his detriment. the reliance piece can be missing with charities, but not the writing!

* An avid runner was diagnosed with a serious heart condition. The runner’s doctor advised her to avoid strenuous physical activity, including running, as such activity would create a substantial risk of cardiac arrest. The runner refrained from such activity for a month, but in that time she gained 15 pounds and felt very unhealthy. Deciding that the health benefits of running outweighed the risk involved, one morning she set out on her normal running path—the shoulder of a flat rural road. Five miles into the run, the runner suffered a heart attack, collapsed, and lapsed into a coma. Two minutes later, the runner’s feet and legs—which were partially sticking out into the travel lane—were run over by a car. The driver of the car, who had been traveling at a reasonable speed, was aware of the runner but was unable to avoid her due to a locking up of the car’s brakes that the driver had negligently failed to have repaired. The runner survived, but suffered serious injuries to both of her legs. The runner sued the driver for those injuries in a jurisdiction that applies traditional contributory negligence rules. Is the runner likely to prevail?
* Learner Selected Answer: No, because the runner was contributorily negligent.

Answer choice A is correct. In a contributory negligence jurisdiction, the plaintiff’s contributory negligence is a complete bar to the plaintiff’s recovery. Here, the runner was negligent in running against her doctor’s advice. Consequently, despite the driver’s negligence in failing to properly maintain his car, which led directly to the accident, the runner cannot recover from the driver. Answer choice B is incorrect because, although the driver was aware of the runner’s predicament before the accident, the driver could not avoid hitting the runner by acting reasonably. The driver’s negligence (i.e., his failure to repair the car’s brakes) occurred prior to his awareness of the runner’s predicament and could not be corrected after gaining such awareness before the accident. Answer choice C is incorrect because, although the runner was unable to extricate herself from the situation since she was unconscious, the driver may be liable if he had the last clear chance to avoid the accident. However, as explained with regard to answer choice B, the driver did not have the last clear chance to avoid the accident and is not liable. Answer choice D is incorrect because, in a contributory negligence jurisdiction, the negligence of the defendant is not sufficient to permit the plaintiff to recover where the plaintiff is also negligent.

in a contributory negligence jurisdiction when is D liable even though P was also negligent?

last clear chance

* A father and son are charged with burglary. Prior to the trial, the prosecutor approached the son and asked him if he would be willing to testify against his father in exchange for a reduced sentence. After discussing the son’s role in the burglary and some negotiation, the prosecutor and the son’s defense attorney reached a settlement agreement. The son pleaded guilty, and was called to testify against his father. On cross-examination of the son, the father’s attorney brought up the fact that the son was also originally charged with the burglary, and asked whether it was true that he received a lesser sentence for agreeing to testify against his father. The prosecutor objected to this line of questioning. How should the court rule on the prosecutor’s objection?

Answer choice C is correct. While settlement offers and negotiations generally are not admissible for public policy reasons, they are admissible in order to prove the bias or prejudice of a witness. Because the son was offered a reduced sentence in exchange for his testimony, the jury should be able to weigh that fact when considering his credibility. Answer choice A is incorrect because the statement is not hearsay; it is not being introduced to show the truth of the matter asserted, but instead is being introduced to impeach the witness’s credibility. Note, however, that a statement made for the purposes of settlement cannot be admitted as a prior inconsistent statement to impeach a party. Answer choice B is incorrect because, as explained above, even though a settlement offer is not admissible to prove liability, it can be introduced for other reasons, such as to prove the bias or prejudice of a witness (as here), to negate a claim of undue delay, or to prove obstruction of a criminal investigation or prosecution. Answer choice D is incorrect because, as mentioned above, it can be relevant to prove—among other things—bias, which could affect how much weight the trier of fact gives the witness’s statement.

* The owner of a lakefront home in a retirement community that greatly restricts access by nonresidents was aware that her dock needed repair, but was unable to afford the considerable expense to do so. The owner placed a large heavy chair at the entrance to the dock with a sign that read, “Please do not enter. Dock in need of repair.” Two children, a six-year-old boy and a ten-year-old girl, entered the property without permission from, or knowledge of the owner. The children quickly discovered the dock. The girl read the sign aloud to the boy and advised him, “You shouldn’t go out on the dock.” The boy, responding “But it’s not dangerous,” climbed over the chair and walked out onto the dock. As the boy ran to the end of the dock, a rotten plank on which the boy stepped gave way, and he fell into the lake and drowned. As permitted by the applicable jurisdiction, the boy’s parents sued the owner in a wrongful death action alleging that her negligence with respect to the dock caused the boy’s death. At trial, the boy’s parents argued that the dock constituted an attractive nuisance. Which of the following may protect the owner from liability that otherwise would arise under this doctrine?

Answer choice A is correct. In order for the attractive nuisance doctrine to apply, the landowner must know or have reason to know that the artificial condition is located in a place that children are likely to trespass. Because the owner lives in a retirement community that greatly restricts access by nonresidents, this requirement is not satisfied. Answer choice B is incorrect because, despite its name, the attractive nuisance doctrine does not require that the child be enticed onto the property by the presence of the condition. Answer choice C is incorrect because the purpose of the attractive nuisance doctrine is to permit a child trespasser to recover for a landowner’s negligence. Answer choice D is incorrect because in order for the attractive nuisance doctrine to apply, the child, due to his youth, must not appreciate the danger presented by the condition. A warning will often protect a landowner from the liability by bringing the condition to the attention to the child. In this case, despite the warning, the child, while deciding that he should go on the dock, did not realize that the reason he should not do so was its dangerous condition.

* A uniformed police officer learned about a possible burglary of a home and went to investigate. When the officer arrived, she attempted to get into the home through the front door, but found it locked. Going to the back of the home, the officer found a door slightly open. Drawing her gun, she entered the home and announced that she was a police officer. The homeowner, honestly but unreasonably fearing that the officer was the person who had broken into the home earlier, shot and killed the officer. The homeowner was charged with murder of the police officer. The jurisdiction recognizes “imperfect” self-defense. Can the homeowner be convicted of this crime?

Answer choice D is correct. Although a defendant who is not the aggressor is justified in using reasonable force in self-defense against another person to prevent immediate unlawful harm to himself, the defendant’s belief that the other person’s actions represent an immediate threat must be reasonable. When such belief is unreasonable but honest, the defendant is entitled to assert “imperfect self-defense,” which reduces his crime from murder to voluntary manslaughter. Consequently, the homeowner cannot be convicted of murder, since he acted in self-defense on his honest but unreasonable belief that the officer threatened him with death or serious bodily harm. Answer choice A is incorrect because, although the homeowner did kill the officer, his imperfect self-defense reduces the charge from murder to voluntary manslaughter. Answer choice B is incorrect because, while the homeowner’s unreasonable belief of the need to use deadly force prevents him from successfully asserting self-defense, his honest belief of the need to use deadly force permits him to successfully lay claim to the imperfect self-defense. Answer choice C is incorrect because, although an accurate statement of the law, the absence of a duty to retreat protects the homeowner from criminal liability for the use of deadly force only when the homeowner’s belief as to the need to use deadly force was reasonable.

* A defendant in a federal securities case introduced the testimony of a witness who had claimed on direct examination that the defendant had no prior knowledge of a change within a corporation’s executive board; the defendant’s knowledge of this fact was a central issue in the case. The prosecutor did not cross-examine the witness. On rebuttal, the prosecutor called a witness who claimed to have been with both the defendant’s witness and the defendant when the defendant learned of the change in question, and had heard the defendant’s witness say, on more than one occasion, that the defendant knew of the change. Further, the prosecutor introduced a properly authenticated email that the defendant’s witness had sent to the witness containing the same information. The defendant’s attorney objects on the grounds that the testimony of the prosecutor’s witness and the email are inadmissible. Should the court admit the testimony of the prosecutor’s witness and the email?

Answer choice A is correct. A witness may be impeached by showing that the witness has made statements that are inconsistent with some material part of the witness’s testimony. Note that because these statements are being used to impeach the witness and not to prove the truth of the matter asserted, they are not hearsay. Note also that because the statements were not made under oath in a prior proceeding, they may not be considered as substantive evidence. Answer choice B is incorrect because both the testimony and the email are admissible for impeachment purposes. Impeachment with a prior inconsistent statement may occur by cross-examination or by use of extrinsic evidence, such as the testimony of another witness or documentary evidence. Answer choice C is incorrect because, unlike the common law, the Federal Rules do not require that the witness have the opportunity to explain or deny the evidence before introducing a prior inconsistent statement. Answer choice D is incorrect because the facts indicate that whether the defendant possessed the knowledge in question is a central issue in the case—that is, it is a material matter. It is true that extrinsic evidence of a prior inconsistent statement cannot be used to impeach a witness regarding a collateral, or immaterial, matter; when a witness testifies as to an immaterial matter, the questioning party is bound by the answer the witness gives.

* A retail store that specialized in glass objects entered into a written contract to purchase 100 hand-blown glass ornaments from an artisan. Because of the artisan’s popularity, the store paid in full for the ornaments at the time that the contract was executed. The contract specified that the store would pick up the ornaments after notification that they were ready. The contract contained no other terms related to delivery of the ornaments and did not allocate the risk of loss. When the ornaments were ready, the artisan notified the store. The parties arranged for the store to pick up the packaged ornaments no later than 2:00 pm the next day. The employee assigned by the store to make the pickup did not arrive until 6:00 pm. In the late afternoon just before the store employee arrived, a short but intense storm caused a large, healthy tree on the artisan’s property to fall over and destroy all the ornaments. Neither party had insured the ornaments against such a loss. Who bears the risk of the loss with respect to the ornaments?

Answer choice B is correct. The UCC provides that a merchant seller generally retains the risk of loss in the absence of a contract term to the contrary until the buyer receives the goods. However, if the buyer is in breach of the contract, the risk of loss passes to the buyer to the extent of any deficiency in the seller’s insurance coverage. Here, the store, as buyer, was in breach of the contract by failing to pick up the ornaments by 2:00 pm. Although the UCC only requires that the delivery time be “reasonable” in the absence of a specific contract term, the parties here modified the contract in that regard by agreeing that the seller should pick up the ornaments by 2:00 pm. Consequently, answer choice B is correct and answer choice C is incorrect. Answer choice A is incorrect because, although the risk of loss passes to the buyer upon tender of delivery of the goods when the seller is not a merchant, the artisan here is a merchant (he has specialized knowledge or skill peculiar to glass ornaments). Consequently, the risk of loss does not pass until the buyer receives the goods unless the buyer is in breach of the contract (as was the case here). Answer choice D is incorrect because, although the store, as buyer, was a merchant with respect to the ornaments, this status is irrelevant to issue of risk of loss. It is the seller’s status as a merchant that can delay the shift in the risk of loss from the tender of delivery by the seller to the buyer’s actual receipt of the goods.

when someone who suffers from epilepsy is told they can't drive under a statute that bars an epileptic from driving and they drive and cause an accident due to their negligence, but they were not having a seizure at the time, can they be held liable under negligence per se?

no. the statute in this fact pattern intends to protect people from a driver having a seizure, not from the guy's other negligent behavior

* In a murder trial, the prosecutor plans to call an eyewitness to the stand to testify that he saw the defendant kill the victim. However, the witness recently suffered a severe head injury that seriously affected his memory. The witness can no longer remember witnessing the murder. Prior to the witness’s injury, he testified to what he saw before the grand jury. The prosecutor would like to introduce the witness’s grand jury testimony as substantive evidence that the defendant committed the murder. The defendant objects to the introduction of the evidence. Should the court admit the witness’s grand jury testimony into evidence?
* Learner Selected Answer: Yes, if used to refresh the witness’s recollection.

Answer choice D is correct. Although the witness is “unavailable” for the purposes of the hearsay rules (as will be discussed below), and there is a “former testimony” exception to the hearsay rule, the former testimony exception does not apply to grand jury testimony. Although grand jury testimony could be admissible as a prior inconsistent statement, because the witness is not testifying here, there is no statement that is “inconsistent” with a prior statement. To be admissible, the former testimony of an unavailable witness must be given under oath in a hearing or deposition, and the party against whom the testimony is being offered must have had an opportunity and similar motive to develop the testimony by direct or cross-examination; grand jury testimony does not meet this standard because the defendant does not have the opportunity to cross-examine grand jury witnesses. Answer choice A is incorrect because, as discussed above, even though the witness is unavailable to testify, grand jury testimony does not fall within the former testimony exception. Answer choice B is incorrect because items used to refresh the witness’s recollection are not admitted into evidence and also because the witness has no recollection that can be refreshed. Answer choice C is incorrect because the witness does meet the standard for “unavailability.” A witness is considered unavailable if that person is exempt on the grounds of privilege, refuses to testify, lacks memory of the subject matter of the statement, is unable to testify due to death or physical or mental disability, or is absent and cannot be subpoenaed or otherwise made to appear. The witness’s lack of ability to remember the subject of his testimony due to a brain injury qualifies the witness as unavailable.

* In the Labor Management Relations Act, Congress expressly authorized the president to seize plants to avert a labor shutdown if the president determined that a shutdown would threaten national security. In response to a threatened national strike by America’s steel workers, the president ordered the government to seize and operate steel mills to ensure steel production that the president deemed vital to the War on Terrorism and hence to national security. Subsequent to the order, Congress did not explicitly approve or disapprove of the president’s action. One of the companies affected by the president’s order filed a suit in an appropriate federal court claiming that the order violated the Constitution. What is the most likely ruling?

Answer choice C is correct. Presidential power under Article II with respect to domestic matters is greatest when the president acts pursuant to Congressional authorization. Here, Congress expressly authorized the president to seize plants to prevent a shutdown that would threaten national security. Answer choice A is incorrect because, although the delegation of legislative power to the executive is subject to the requirement that the exercise of such power must be subject to a specific, intelligible standard, the Supreme Court has so loosely interpreted this requirement that almost any standard has satisfied this requirement. Answer choice B is incorrect because, although the president’s power as Commander-in-Chief may not extend to this specific domestic action, it is not unconstitutional because it has been authorized by Congress. Answer choice D is incorrect because action by the president in domestic affairs that has been authorized by Congress need not be subsequently approved by Congress in order to be valid.

* A recidivism statute calls for a mandatory life sentence for a defendant who is convicted of three felonies. The defendant was convicted of felony theft three separate times and was sentenced to life in prison after his conviction for the third theft. In each case, the defendant stole the items from stores when nobody was watching. He did not use any weapons, nor was he violent. The defendant challenges the sentence on constitutional grounds. Will the defendant succeed?

Answer choice D is correct. The Eighth Amendment prohibition does not prohibit life sentences for three-time repeat felony offenders, even if they are non-violent, making answer choice A incorrect. Answer choice B is incorrect because this type of recidivism statute has been judged not to violate the Double Jeopardy Clause. Answer choice C is incorrect because the Eighth Amendment applies to all cruel and unusual punishment, not just that which involves the use of force. Some sentences that are not proportional to the crime have been held to violate the Eighth Amendment even if the sentences do not involve the use of force.

does the parole evidence rule bar all evidence of an oral agreement between parties who have entered into a written contract?



what type of evidence does it absolutely bar, even when the contract is only partially integrated?

the parol evidence rule does not bar all evidence of an oral agreement between parties who have entered into a written contract. For example, the parol evidence rule does not apply when a party is raising an excuse, establishing a defense, evidencing a separate deal, proving a condition precedent, clarifying an ambiguity, proving subsequent agreements, or making certain clarifications under the UCC.



evidence that of a term of a prior agreement that contradicts a term that's in the final written contract.

* At trial, in an effort to prove that the defendant suffers from a mental defect, a criminal defense attorney seeks to introduce evidence that his client told several people that he believed he was the President of the United States. The prosecutor contends that the evidence is inadmissible. Is evidence of the defendant’s statement admissible?

Answer choice C is correct. A statement offered as circumstantial evidence of the declarant’s mental state is admissible as nonhearsay. Answer choice A is incorrect because this statement is not offered to prove the truth of the matter asserted (i.e., that the defendant is the president), but to support the defense attorney’s assertion that the defendant has a mental defect. Answer choice B is incorrect because, as the statement is not hearsay, it does not need to fall under a hearsay exception; there is a distinction between the “state of mind” exception to the hearsay rule and using a statement as circumstantial evidence for a declarant’s state of mind. Answer choice D is incorrect because while defendants in a criminal trial occasionally are subject to different evidentiary standards than the prosecution, they are almost always subject to the same hearsay rules.

An indigent defendant was indicted for driving under the influence of alcohol, a misdemeanor. The lawyer who had been appointed to represent the defendant suffered a fatal heart attack on his way to the courthouse on the day scheduled for the trial. In discussing the absence of the defendant’s lawyer with the defendant, the trial judge learned from the defendant that he intended to plead guilty. The judge indicated that, in exchange for the defendant’s guilty plea, the defendant would not serve time in prison. The defendant agreed and was sentenced to two months in prison, with the sentence suspended. The defendant appealed his conviction, contending that he was denied his Sixth Amendment right to counsel at trial. Should the judge reverse the defendant’s conviction?


Answer Choices: 1. No, because the defendant was convicted of a misdemeanor.
2. No, because the defendant’s prison sentence was suspended.
3. Yes, because the defendant was convicted of a crime for which a sentence of incarceration was imposed.
4. Yes, because the defendant was convicted of a crime that was punishable by imprisonment.

Answer choice C is correct. A defendant has the right to the assistance of counsel at any trial that results in incarceration, even when that sentence is suspended. Answer choice A is incorrect because even a misdemeanor that results in a sentence involving incarceration requires the assistance of counsel. Answer choice B is incorrect because the fact that the sentence was suspended does not waive the requirement that defense counsel be provided. Answer choice D is incorrect because assistance of counsel is not required for a crime that is punishable by imprisonment as long as that punishment is not imposed.

when might an executive branch order that amounts to a prior restraint on free speech be upheld?

when there is an imminent threat to national security (ie a terrorist group is going to use US newspapers to send out coded messages and POTUS orders the papers shut down for two weeeks till the threat has been neutralized) - there's no requirement that the papers be compensated

* A federal statute provides that “all persons within the United States shall have the same right in every state to make and enforce contracts as is enjoyed by white persons.” The Supreme Court interpreted this statute as applying to all contracts, including private contracts. A black citizen of a state in the United States claims that an appliance store in her state violated this statute by refusing to enter into a sales contract with her because of her race. The appliance store defended on the ground that the statute is unconstitutional. A federal court would be most likely to uphold this statute by relying upon which provision of the Constitution?

Answer choice A is correct. Unlike the Fourteenth Amendment and the Contracts Clause, the Thirteenth Amendment does not require state action. Rather, it abolishes slavery and its “badges and incidents,” including racial discrimination in private transactions like contracts. Moreover, Section 2 of the Thirteenth Amendment authorizes Congress to enact legislation to implement its guarantees. Answer choices B and D are incorrect because, as mentioned, the Contracts Clause and the Fourteenth Amendment protect against wrongful conduct by the government, rather than a private party such as the appliance store. Answer choice C is incorrect because the General Welfare Clause permits Congress to exercise its spending and taxing powers for any public purpose, but it does not create a specific power to legislate for the public welfare in general.

when may a wife testify about finding incriminating evidence (documents) that will help the prosecutor convict her husband of a crime?

In federal court, when she wants to! defendant spouse can't stop her.



note, this isn't "confidential marital communication" in which both spouses hold the privilege, it's spousal immunity, which is held by the wife (the non-defendant spouse) in federal court (in state courts, the rule is usually that the prosecution can't call a spouse as a witness against the Defendant



does the doctrine of part performance bar me from backing out of a contract to buy land even if I'm the one who partially paid and began constructing something on the land? (I'm the buyer and I improved the land and now I want to back out?)

yup. it just proves that we intended to enter into a contract

* A fisherman who lived next to a lake owned a large sport-utility vehicle equipped with a trailer hitch. He used the vehicle primarily to tow his large fishing boat. One afternoon, a neighbor asked if she could borrow the fisherman’s vehicle for a short time in order to tow her boat back from the dock, as her car was at the repair shop. The fisherman agreed to let the neighbor use the vehicle to tow her boat, but asked her to return the vehicle immediately afterward. The neighbor drove the vehicle to the dock and towed her boat back without incident. Before returning the vehicle, the neighbor decided to buy a gift for the fisherman as a token of appreciation. While the neighbor was driving the vehicle to the store to buy the gift, she was involved in a serious accident. The neighbor was not seriously hurt, but the vehicle was a total loss. If the fisherman sues his neighbor for conversion, will he prevail?

Answer choice A is correct. Consent is effective if a plaintiff, by words or actions, manifests the willingness to submit to a defendant’s conduct, but the defendant’s conduct may not exceed the scope of the consent. Here, the fisherman clearly consented to his neighbor’s use of the vehicle to tow her boat; however, by using the vehicle a second time, the neighbor exceeded the scope of the fisherman’s consent. Thus, she is liable for conversion. Answer choice B is incorrect because, while the concept of frolic and detour may be applicable to determine an employer’s vicarious liability for an employee’s action, it is not applicable in a non-employment situation. Answer choice C is incorrect because once the scope of consent has been exceeded, the purpose of the defendant’s conduct is irrelevant. Answer choice D is incorrect because, as discussed, the neighbor exceeded the scope of the fisherman’s consent.

* Congress enacted the Health Care Act (HCA) “to ensure all Americans access to health care at a reasonable cost.” Congress delegated to an executive agency, the Department of Health & Human Services (HHS), responsibility for promulgating regulations to implement the HCA. The HCA further provided that a joint House/Senate committee can repeal or revise the HHS regulations if the committee determines that they inadequately fulfill the HCA’s purpose. Would a court be likely to hold that the HCA is unconstitutional?

Answer choice B is correct. Congress cannot authorize a legislative committee to “veto” agency regulations. The Constitution requires that legislative power be exercised in accord with bicameralism (i.e., passed by both Houses of Congress) and the Presentment Clauses of Article I (i.e., the requirement that, in order for a bill to become law it must be presented to the president for approval or return). Answer choice A is incorrect because Congress is not prohibited from delegating legislative power to an executive agency so long as Congress specifies intelligible standards for the agency to follow. Answer choice C is incorrect because, although Congress is reasonably trying to vindicate its Article I legislative power by ensuring the accountability of executive agencies that make law, the chosen means constitute an unconstitutional legislative veto. Answer choice D is incorrect because, although the joint committee action would constitute the exercise of legislative rather than executive power, the action would not comply with the constitutional requirements.

voluntary intoxication is a defense to what kind of crime? if it does what?

specific intent crime - only if it negates the intent element of the crime

* A defendant on trial for forging checks took the stand in his own defense. On direct examination, the defendant denied having forged any checks; he stated that before he graduated from college the year before, he worked in his university’s academic records office, indicating that he was “a trustworthy person.” On cross-examination, the prosecutor asked the defendant if he had falsified records while working in the academic records office. The defendant denied that he had done so. The prosecutor then wanted to call to the stand his former supervisor from the university to testify that she had to investigate the defendant after allegations of misconduct, and that when questioned, he had admitted to her that he had falsified records. The defendant was removed from his position, but no formal charges had been brought against him. Should the prosecutor be allowed to call the defendant’s former supervisor to the stand to testify as to the falsified records?

Answer choice D is correct. The former supervisor may not testify about the falsified records because it would be impeachment by extrinsic evidence of a prior bad act. A prior bad act, if used to impeach the credibility of a testifying witness, may not be proved by the introduction of extrinsic evidence. The adverse party may cross-examine the witness about the prior bad act, but must take his answer as he gives it. Since the evidence about the falsification of records is only admissible, if at all, to impeach the witness, extrinsic evidence—such as the supervisor’s testimony—may not be used to refute the defendant’s denial. Answer choice A is incorrect because when a person is charged with one crime, extrinsic evidence of a prior bad act is inadmissible to establish that the defendant had a propensity to commit that crime. Since the facts do not indicate that the prior bad act is being used as evidence for something circumstantial and relevant, such as motive, common plan or scheme, or identity, the supervisor’s testimony is not admissible as substantive evidence. Answer choice B is incorrect because, for the reasons listed above, extrinsic evidence is not admissible to prove a witness’s prior bad act. Answer choice C is incorrect because the alleged hearsay statement is a statement by an opposing party, and therefore nonhearsay; further, it would constitute a prior inconsistent statement.

* A patron at a resort ranch took part in a supervised horseback trail ride. Prior to the ride, the patron executed a valid release that enumerated the inherent risks of horseback riding and, by its terms, relieved the resort from liability from any loss, damage, or injury to the guest’s person or property suffered during the ride attributable to the negligence of the ranch or its employees. The patron was injured by a fall from the horse. The horse reared in response to negligent behavior of another rider who was also a patron at the ranch. The patron filed suit against the ranch and the other rider for damages resulting from his injuries that totaled $400,000. At trial, it was determined that the ranch was 75% at fault for the patron’s injuries due to its selection and training of the horse, and that the other rider was 25% at fault. The applicable jurisdiction recognizes the validity of such releases and has enacted both a modified comparative negligence statute and a pure several liability statute. How much can the patron recover from the ranch?

Answer choice A is correct. In general, parties can contract to disclaim liability for negligence. An exculpatory provision in a contract acts as a bar to recovery for harms arising from the negligence of the party protected by the contract. This bar applies even where the state has adopted a comparative negligence statute. Here, the patron entered into a valid agreement to exculpate the ranch from liability for its negligence. Consequently, answer choices B, C, and D are incorrect.

A concert violinist received an offer by mail to play a concerto with a local symphony orchestra. She checked her schedule, and thinking that she had the date free, mailed a letter to the symphony orchestra accepting the offer. Later that day, as she was checking her calendar about another matter, she realized that she had a rehearsal for another performance on that date. The violinist called the orchestra manager and declined the offer to play the concerto. In a breach of contract action by the orchestra against the violinist, will the orchestra prevail?

yes - mailbox rule

* A defendant was convicted of bank robbery in federal court. Subsequently, the defendant was indicted in the state where the bank was located for the crimes of robbery and conspiracy to commit robbery. The defendant moved to dismiss the state prosecution of these offenses on double jeopardy grounds. Should the defendant’s motion be granted?

Answer choice D is correct. Under the “Dual Sovereignty” doctrine, prosecution of a defendant by the federal government for a crime arising out of an event does not prevent a state from prosecuting the defendant for a crime arising out of the same event. (Note: Under this doctrine, the reverse is also true.) Under Blockburger, robbery and conspiracy to commit robbery are separate offenses. Each contains an element that the other does not. Consequently, prosecution of the defendant for either robbery or conspiracy to commit robbery by the state is not prohibited by double jeopardy. For these reasons, answer choices A, B, and C are incorrect.

* On trial for second-degree murder, the defendant pled not guilty, asserting that he acted in self-defense. The statute in the jurisdiction defines second-degree murder as “the intentional killing of another human being with malice and without provocation.” The jurisdiction also has a statute stating that “all affirmative defenses are to be proved by the defense, and the burden of persuasion shall be by a preponderance of the evidence.” The judge instructed the jury that the self-defense evidence presented by the defendant should not be considered by the jury for any purpose unless the jury first determined that this evidence satisfied the “preponderance of the evidence” standard. The jury found the defendant guilty of second-degree murder. The defendant appealed his conviction, contending that the state statute and the jury instructions violated his constitutional rights. How should the appellate court rule on this appeal?

Answer choice C is correct. A state (or the federal government) may place the burden of proving an affirmative defense, such as self-defense, on the defendant without violating the Due Process Clause. The preponderance of the evidence standard for judging whether the defendant has met this burden is also constitutional. The jury instructions, however, improperly prevent the jury from considering the defendant’s self-defense evidence, not as an affirmative defense, but as a defense to the elements of the crime that the prosecution must prove. Used in such a manner, this evidence need not satisfy the preponderance standard before being considered by the jury. For these reasons, answer choices A, B, and D are incorrect.

what is an exception to the "commandeering clause?"

The Elections Clause of Art. I explicitly empowers Congress to override state laws concerning federal elections. This express provision makes irrelevant general principles of federalism embodied in the “commandeering” cases

* During a trial for attempted murder, the prosecutor seeks to introduce into evidence the victim’s properly-authenticated emergency room report. The report describes the victim’s stab wounds and treatment. The report also includes a statement that the victim made to his doctor during a check-up the following day, naming the defendant as his assailant. The prosecutor wants to introduce the record to prove the extent of the victim’s injuries, and as evidence that the defendant was responsible for the victim’s harm. Is the victim’s emergency room report admissible?

Answer choice A is correct. The victim’s statement to the doctor is hearsay within hearsay, which means that both levels of hearsay must fall within a hearsay exception in order to be admissible. The medical record itself falls within the business records exception; medical records are considered business records to the extent that they relate to diagnosis or treatment. Statements related to fault associated with the injury are not admissible under this exception. The victim’s statement would have to fall under its own hearsay exception, and none is applicable here, which makes answer choice B incorrect. Answer choice C is incorrect because the statement was not made for the purpose of medical treatment; while statements of fault can be admissible if reasonably pertinent to diagnosis or treatment, this statement was made during a check-up after the patient already had received treatment, and therefore was not likely necessary for diagnosis. Answer choice D is incorrect because the declarant’s availability is immaterial to the business records exception. Further, the victim would not be the declarant of the record itself; some member of the emergency room’s personnel would be.

If a dealer promises to sell me a car for $35,000, and we write up the deal, then the car goes up in price with the manufacturer and the car dealer talks me into orally agreeing to pay $37,000, can I change my mind about not paying $2000 more?



does the preexisting duty rule apply to the requirement to get the modification in writing?



is there an exception?

yes - the modification was not in writing (required under the SOF for modification of a contract for goods over $500).



not in UCC world



yes - in the goods are specialty goods and not generally marketable (the car isn't, even though it was ordered to my specs, it's generally marketable)

* A buyer purchased a motor home from a private seller. After taking possession of the motor home, the buyer discovered that the bedroom of the motor home was infested with bed bugs, and pest control treatments were unsuccessful in eradicating the problem. The buyer honestly claims that he would not have purchased the motor home had he known of the infestation. At the time of the sale, the seller knew of the infestation but did not disclose the condition to the buyer. When the buyer commented to the seller at the time of the sale that the buyer assumed that the motor home did not have bed bugs, the seller simply did not respond. The buyer was justified in relying on the seller’s silence as an assertion that the mobile home did not contain bed bugs. The seller’s actions violated her duty of good faith and fair dealing. What is the best description of the status of the contract between the buyer and the seller?

Answer choice A is correct. Nondisclosure of a known fact is tantamount to an assertion that the fact does not exist if the party not disclosing the fact knows that disclosure would correct a mistake of the other party as to a basic assumption, and the failure to disclose would constitute lack of good faith and fair dealing. Here, the seller’s nondisclosure affected a basic assumption of the buyer (i.e., that the mobile home did not have bedbugs) and the seller’s actions violated her duty of good faith and fair dealing. The misrepresentation was fraudulent because the seller knowingly and intentionally made a false assertion about an element of the contract in order to cause the assent of the buyer, who justifiably relied on this assertion. Thus, the contract is voidable by the buyer, making answer choice C incorrect. Answer choice B is incorrect because the seller has no right to void the contract. Answer choice D is incorrect because a contract is void only in instances of fraud in the factum—where a fraudulent misrepresentation prevents a party from knowing the character or essential terms of the transaction.

if I am the future interest holder of an estate, I have the right to inspect the premises for waste. what is this called?

I have a license and an interest in the estate.

* A woman took her car to an unscrupulous auto mechanic’s garage for a tune-up. The woman’s car had a new and expensive set of tires that the mechanic coveted. The woman left her car at the garage overnight. Later that night, after the woman had left the premises, the mechanic took the tires off the woman’s car, put them into a back room of his garage, and replaced the tires with a cheap, old set. That same evening, the woman’s friend told her about the mechanic’s unscrupulous nature, and that he had a habit of stealing tires. The woman went back to the garage the next morning. Noticing that the tires on her vehicle were different, she demanded that the new, expensive tires be put back on the vehicle. The mechanic complied, and the woman left the premises. The woman reported the mechanic to the police, and the mechanic is charged with larceny. Based on the foregoing facts, should he be convicted of the crime?

Answer choice C is correct. For a larceny, the initial taking and asportation of another’s property must be trespassory; that is, the defendant must not be legally entrusted with the property. Here, the woman entrusted the mechanic with her vehicle (and the tires on the vehicle). Thus, the initial taking of the tires was not trespassory, and the mechanic’s crime was embezzlement, not larceny. Answer choices A and B are incorrect because, while both correctly apply elements of larceny, they incorrectly state that the mechanic should be convicted. Because the mechanic had the intention of permanently depriving the woman of the tires at the time of the taking, answer choice D is incorrect. That he later returned them to the woman is of no matter.

* A man and his friend attended their 10-year high school reunion party. There, the two struck up a conversation with a woman who had been a classmate. Neither the man nor his friend had seen her since high school. At the end of the reunion party, the three decided to walk to a nearby bar. As they were walking to the bar, the friend suggested a shortcut through an alley. In the alley, the friend grabbed the woman and began making unwanted sexual advances towards her. The man, despite the woman’s pleas to help her, continued walking on towards the bar. Once there, the man ordered a beer and watched a sporting event on television, while his friend raped the woman in the alley. The man was charged as an accomplice to rape. Should he be convicted of the crime?

Answer choice A is correct. The man owed no duty to the woman, since he had no special relationship with her. Consequently, his failure to act to prevent his friend from raping her does not constitute an actus reus on which criminal liability can be predicated. Answer choice B is incorrect because accomplice liability, unlike conspiracy, does not require an agreement between the parties. Answer choice C is incorrect because, although the man failed to take any action to stop his friend from raping the woman or even to report the crime, he did nothing to encourage or to assist the friend in committing the rape. Answer choice D is incorrect because mere awareness that a crime is occurring is not sufficient to trigger accomplice liability.

* Based on an advertisement in a local newspaper, a state resident bought a cross-country roundtrip ticket on a national airline for $450. The ad did not mention that the airline charged $75 for any changes to a ticket. Because of illness, the state resident had to change her return flight, and the airline charged her $75. The state resident refused to pay, citing a state law that required any ad for the sale of tickets for any event or trip to clearly disclose any monetary penalties for changing tickets. The airline sued the state resident in federal court for the unpaid fee, arguing that the state law is invalid, citing a federal statute prohibiting states from enforcing any law “relating to the rates, routes, or services” of any airline. Will the airline prevail?

Answer choice B is correct. The language of the federal statute indicates a specific intent by Congress to preempt the field of airline regulations with regard to rates, routes, and services. Under the Supremacy Clause, federal law trumps conflicting state law. Answer choice A is incorrect because commercial speech is subject to content regulation. The Freedom of Press Clause does not grant the press greater free speech rights than the public at large enjoys. Answer choice C is incorrect because, although there is a judicially recognized presumption against preemption with respect to a conflict between state and federal law, this presumption is rebuttable. Answer choice D is incorrect because, although the state law does not purport to deal specifically with interstate commerce in general or airline travel specifically, there is clear conflict between the federal statute and the state law with respect to the fee charged by the airline.

* Four men were bow-and-arrow hunting in a thickly wooded area. Each man was wearing brightly-colored apparel to minimize the risk of an accident. However, midway through the day, one hunter was struck in the leg by an arrow. The wounded man collapsed in agony, and eventually required several surgical procedures and months of rehabilitation in order to walk again. None of the other three men admitted responsibility for the accident. The injured man sued the other three hunters, claiming negligence. The injured man (i.e., the plaintiff) introduced evidence tending to show that no other hunting parties were within a five-mile radius at the time of the incident, but he was unable to show conclusively which of the three defendants fired the arrow that caused his injury. At the conclusion of the plaintiff’s case, one of the defendants filed a motion for summary judgment. How should the court rule on the motion?

Answer choice C is correct. A plaintiff has the burden of proving by a preponderance of the evidence that each defendant’s actions were an actual and proximate cause of the claimed injury. The plaintiff here failed to establish that the defendant who moved for summary judgment was negligent. Consequently, the defendant is entitled to summary judgment. Answer choice A is incorrect because exoneration would be required only if the plaintiff could establish that all of the defendants had been negligent, in which case each one would have to exonerate himself. If the plaintiff’s harm was caused by one of a small number of defendants, each of whose conduct was tortious (negligent), and all of whom are present before the court, then the court may shift the burden of proof to each individual defendant to prove that his conduct was not the cause-in-fact of the plaintiff’s harm. Here, however, the plaintiff cannot establish that all of the defendants were negligent; rather, only one defendant was, and the evidence does not establish which one. Answer choice B is incorrect because res ipsa loquitur is applied only in cases where the defendant was in exclusive control of the instrumentality that caused the plaintiff’s harm, or at least was responsible for those with such control. Here, the plaintiff has not established who was in control of the arrow that caused his injury. Answer choice D is incorrect because the facts are unclear as to whether the defendant was or was not a substantial factor in the plaintiff’s injury.

* A defendant on trial for battery arising from a barroom brawl sought to introduce the testimony of his grandmother, who would testify that the defendant had a reputation in her church community for being a “helpful and trustworthy person.” Further, the grandmother would offer her testimony regarding an incident that took place when the defendant was 13 years old wherein he refused to engage in a schoolyard fight with one of his classmates. The prosecution objects to the grandmother’s testimony in its entirety. Should the court allow the grandmother to testify?

Answer choice D is correct. While a criminal defendant is permitted to introduce evidence of his good character as being inconsistent with the crime charged, the evidence must be relevant to the character trait at issue. Here, the defendant’s reputation as being “helpful and trustworthy” is not relevant to the crime charged, which is battery. If the defendant had a reputation in the church community as being nonviolent or peaceful, that likely would be admissible. However, since the proffered character traits have nothing to do with getting in a barroom brawl, they are irrelevant. While the testimony as to the schoolyard incident does speak to the character trait of nonviolence, evidence of a specific act is an inappropriate way to introduce good character evidence. The defendant can offer reputation or opinion testimony by another witness to prove character, but not evidence of a specific act unless character is an essential element of a crime or defense, which is not the case here. Answer choice A is incorrect for the reasons listed above: while the defendant can introduce evidence of his own good character, it must be (i) a character trait relevant to the crime charged and (ii) presented as reputation or opinion testimony, unless character is an essential element of a crime or defense. Answer choices B and C are incorrect because they both admit one of the pieces of inadmissible evidence.

Does a subsequent purchaser of real estate validly assume a mortgage if he does so orally?

yes - assumption agreements are not subject to the SOF!

* A biotech start up firm secured a loan from a private investor to purchase land and to build a laboratory facility on that land with a mortgage on the land and the facility. Subsequently, the firm sold the developed property to a partnership. The deed stated that the partnership took the property subject to the mortgage. Later, the partnership sold the developed property to a corporation. Each deed was properly recorded promptly after its applicable closing. Immediately after closing, the president of the corporation, in exchange for adequate consideration, orally promised the partnership that the corporation would assume the mortgage. For four months, neither the corporation nor the previous owners of the facility made the required monthly payments on the mortgage obligation to the lender. The lender has filed an action against the corporation for the past due amounts. Is the corporation liable for these amounts?

Answer choice C is correct. By assuming the mortgage, the corporation became personally liable for the mortgage loan payments. In general, it is the lender-mortgagee’s choice whether to proceed against the original mortgagor or a transferee who has assumed the mortgage obligation. Answer choice A is incorrect because a transferee who assumes a mortgage obligation is not sheltered from personal liability by a prior transferee’s refusal to assume the mortgage obligation. The shelter principle applies in the context of the recording statute to shelter a transferee who otherwise would not qualify for protection under the statute when a prior transferee did. Answer choice B is incorrect because the Statute of Frauds does not apply to an assumption agreement. Answer choice D is incorrect because, while the holder of a purchase money mortgage is entitled to priority when the proceeds of a foreclosure sale are distributed, the loan’s status as a purchase money mortgage obligation does not affect the liability of a transferee of the mortgage.

* A defendant is on trial for cocaine possession. The cocaine was found during a warrantless search of the defendant’s car by a police officer. The search occurred immediately after the defendant was arrested for driving a car with an inoperative taillight, a misdemeanor punishable only by a fine. The defendant had been placed in a police car prior to the search. The cocaine was found inside a closed bag on the back seat of the passenger compartment of the defendant’s car. The defendant now moves to suppress the cocaine. Will the defendant’s motion be granted?

Answer choice A is correct. In order to qualify as a lawful search incident to arrest, a search of a car in which the defendant was an occupant must be made at the time that the defendant has access to the car or to uncover evidence of the crime for which the defendant was arrested. Here, neither of those circumstances is applicable. Consequently, answer choice D is incorrect. Answer choice C is incorrect because, although the automobile exception permits a police officer to search a car without a warrant, the exception only applies when the police officer has probable cause to conduct the search. Answer choice B is incorrect because a police officer’s arrest of an individual for a misdemeanor that is punishable only by a fine is not unreasonable under the Fourth Amendment.

* At the defendant’s trial on a narcotics charge, the prosecution introduced the former testimony of a co-conspirator who had testified against the defendant at a preliminary hearing; the evidence was sufficiently corroborated. The co-conspirator has since fled the country to a jurisdiction with no extradition treaty with the United States. The defendant’s attorney now seeks to impeach the credibility of the co-conspirator. Which of the following types of evidence is the court most likely to admit?

Answer choice A is correct. When a hearsay statement is admitted into evidence, the credibility of the declarant may be attacked by any evidence that would be admissible if the declarant had testified as a witness. In this case, if the declarant co-conspirator had testified, the defendant’s attorney would have been able to cross-examine him as to his bias against the defendant. Because a witness may be influenced by his relationship with a party or in the outcome of a case, evidence of the witness’s bias is always material. Here, the co-conspirator had an incentive to testify against the defendant in exchange for a better deal from the prosecutor. This is relevant and an appropriate way to impeach the witness. Answer choice B is incorrect because misdemeanor convictions are admissible to impeach a witness only if they are probative of untruthfulness. In this case, public intoxication, disorderly conduct, and vandalism have nothing to do with the witness’s character for truthfulness. As such, the misdemeanor convictions are inadmissible for impeachment. Answer choice C is incorrect because the credibility of a witness may only be attacked by reputation or opinion evidence as to the witness’s character for truthfulness or untruthfulness. The fact that the witness has a reputation for being violent is not probative of truthfulness, and is therefore inadmissible. Answer choice D is incorrect because the conviction is more than 10 years old. A witness may be impeached with evidence that he has been convicted of a crime, subject to certain limitations. Any crime involving dishonesty or a false statement, whether a felony or a misdemeanor, may be used to impeach if it is less than 10 years old. Any crime not involving dishonesty that is less than 10 years old may be used for impeachment only if the crime is a felony. If more than 10 years have elapsed since the conviction, evidence of the crime is not admissible unless the court determines that the probative value substantially outweighs its prejudicial effect. In this case, nothing indicates that the felony’s probative value would be substantial in any way. The conviction does not involve dishonesty, and it is over 10 years old. As such, it is inadmissible for impeachment purposes.

* An adult woman was vacationing at a friend’s house on a lake. One afternoon, the woman watched her friend maneuver his motorized personal watercraft around the lake; the friend took a particularly violent spill that temporarily knocked the wind out of him but left him otherwise unharmed. The next morning, without the friend’s knowledge, she decided to take the personal watercraft out on the lake herself. Due to her inability to control the vehicle, it flipped over. As a consequence, the woman suffered serious physical injuries. The woman brought a lawsuit against the friend to recover damages for her injuries. The applicable jurisdiction has adopted comparative negligence rules. Prior to the submission of the case to the jury, the friend requested that the court specifically instruct the jury on the assumption of the risk defense. Should the court grant this request?

Answer choice D is correct. In a comparative negligence jurisdiction, assumption of the risk is not recognized as a separate defense—it has been merged into the comparative fault analysis and merely reduces recovery. The plaintiff’s awareness of the risk of her conduct is generally taken into account in determining the degree to which she is at fault, but it can also be considered in determining the reasonableness of the plaintiff’s or the defendant’s actions. Answer choice A is incorrect because, even assuming that the woman did voluntarily assume that risk, such a defense is not recognized as a separate defense in a comparative negligence jurisdiction. Answer choice B is incorrect because assumption of the risk is only an absolute bar to recovery in a contributory negligence jurisdiction, not a comparative negligence jurisdiction. Answer choice C is incorrect because assumption of the risk requires the plaintiff, not the defendant, to be aware of the risks of the plaintiff’s conduct.

* Concerned about problems caused by overpopulation, a state legislature enacted a statute imposing criminal penalties on any person who is the biological parent of more than two children. The stated purpose of the statute was to preserve the state’s natural resources and improve the quality of life for the state’s residents. After the statute took effect, a married couple who already had two children conceived a third. After the wife gave birth to this child, the couple was arrested and convicted under the statute. Which of the following is the strongest argument for voiding their convictions?

Answer choice B is correct. The statute violates the fundamental liberty and privacy interests of married couples under the substantive aspect of the Due Process Clause. Answer choice A is incorrect because there is a rational basis for the statute. By criminalizing the parenting of more than two children, the statute would discourage some people from having children and thereby address the problem of overpopulation and help to preserve the state’s natural resources. In addition, since the statute affects fundamental rights, the appropriate standard for judging its constitutionality is strict scrutiny rather than rational basis. Answer choice C is incorrect because the statute does not grant to a prosecutor the right to decide who can have children. The statute clearly subjects a man who fathers or a woman who gives birth to more than two children to criminal penalties. Answer choice D is incorrect because individuals who have more than two children are not members of a suspect classification. Consequently, the statute needs only to satisfy the rational basis standard under an Equal Protection analysis, so while this answer is true, it's not the strongest argument.